Neuro Flashcards

1
Q

What are the functions and characteristics of the cornea?

A

Transmission of light

  • Refraction
  • Must be transparent
  • Must have a smooth spherical surface
  • Dehydrate endothelium (no repair)
  • the innermost layer of the cornea, maintain corneal clarity by pumping water out (since water molecules alter the regular spacing between collagen fibres & cause opacity)
  • Surface epithelium (capable of repair)
  • outer layer of the eye, many layers that slough off and are constantly regenerated
How well did you know this?
1
Not at all
2
3
4
5
Perfectly
2
Q

What are the functions and characteristics of the sclera?

A

Forms the white capsule around the eye, except at its anterior surface where it is specialised into the clear cornea

  • Offers protection; formed of a tough outer layer of collagen - Serves as an insertion point for the external muscles of the eye
  • Continuation of dura mater and cornea
How well did you know this?
1
Not at all
2
3
4
5
Perfectly
3
Q

What are the functions and characteristics of the iris?

A

Specialised section of choroid

  • Contains & controls the size of the pupil - which lets light in
  • Sphincter muscles (circular fibres) make the pupil smaller [parasympathetic]
  • Dilator muscles (radial fibres) make the pupil larger [sympathetic]
  • Gives eyes their colour
How well did you know this?
1
Not at all
2
3
4
5
Perfectly
4
Q

What are the functions and characteristics of the ciliary body?

A

Glandular epithelium

  • produces; aqueous humour & nutrients for cornea & lens
  • Aqueous humour: mainly water & electrolytes, located in the anterior chamber important in maintaining intraocular pressure (15mmHg)
  • Made of smooth muscle, which controls accommodation; the adjustment of the lens in the eye so that clear images of objects at different distances are formed on the retina
  • Receives innervation from the parasympathetic system
How well did you know this?
1
Not at all
2
3
4
5
Perfectly
5
Q

What are the functions and characteristics of the choroid?

A

Important for the nutrition of the outer retina (photoreceptors)

  • Acts as a heat sink
  • Darkly pigmented so that it can absorb stray photons
How well did you know this?
1
Not at all
2
3
4
5
Perfectly
6
Q

What are the functions and characteristics of the retina?

A

Layered structure

  • Produces vitreous humour: which acts as a collagen scaffold, helps maintain intraocular pressure and is important in the transmission of light
  • Light passes through the pupil from the visual field to project an image onto the retina. An object that attention is focused on, projects an image that is centred near the posterior pole of the eye along the visual axis, this point is known as the FOVEA CENTRALIS and the surrounding 1cm is known as the MACULA LUTEA at these points the retina is specially modified for maximal visual acuity (resolving power) - Medial to the macula is a region where retinal axons accumulate to leave the eye this is the optic disc (where the optic nerve forms) - photoreceptors are absent in this region so its called the blind spot
  • Retinal pigement epithelium (RPE)
  • Contains photoreceptors (rods & cones) so it is able to convert light into electric impulses (PHOTOELECTRIC TRANSDUCER) which are transmitted to ganglion cells which go on to make optic fibres and eventually the optic nerve
  • Rods: important to vision in dim lighting - very sensitive to light, also important for peripheral vision
  • Cones: important for colour vision
How well did you know this?
1
Not at all
2
3
4
5
Perfectly
7
Q

What are the three layers of tear film and where do they come from?

A

Anterior Lipids (oils): secreted by meibomium glands, provides a hydrophobic barrier to prevent the aqueous layer evaporating

Middle aqueous (water,electrolytes & proteins): secreted by lacrimal glands, regulates transport through the cornea and prevent infection

Posterior mucous:- secreted by goblet cells, provides a hydrophilic layer that allows for the even distribution of the tear film

How well did you know this?
1
Not at all
2
3
4
5
Perfectly
8
Q

Layers through which a photon must travel through the eye?

A
  1. Tear film (transmission)
  2. Cornea (transmission & refraction (contributes to 2/3rds of refraction)
  3. Aqueous humour (transmission)
  4. Lens (transmission & refraction)
  5. Vitreous humour (transmission)
  6. Ganglion cell (transmission)
  7. Amacrine cell (transmission)
  8. Bipolar cell (transmission)
  9. Horizontal cell (transmission)
  10. Cone (transduction)
  11. Rods (transduction)
  12. Pigmented epithelium (absorption of excess photons)
How well did you know this?
1
Not at all
2
3
4
5
Perfectly
9
Q

What are the branches of the internal carotid artery in relation to the eye?

A
Opthalmic artery
Central retinal artery - which passes into the optic nerve
Ciliary arteries
 Lacrimal artery
Ethmoid 
 Eyelid artery
How well did you know this?
1
Not at all
2
3
4
5
Perfectly
10
Q

What are the branches of the external carotid artery in relation to the eye?

A

Facial artery - supplies medial lid & orbit

How well did you know this?
1
Not at all
2
3
4
5
Perfectly
11
Q

What is the effect of damage to the left optic nerve?

A

no vision through the left eye

How well did you know this?
1
Not at all
2
3
4
5
Perfectly
12
Q

What is the effect of damage to the optic chiasm?

A

Loss of vision of the temporal visual fields - this is called hemianopia (since half/hemi of vision has been lost)

How well did you know this?
1
Not at all
2
3
4
5
Perfectly
13
Q

What is the effect of damage to the left optic tract?

A

Loss of vision of the temporal field of the left eye & the loss of the nasal field of the right eye - another type of hemianopia

How well did you know this?
1
Not at all
2
3
4
5
Perfectly
14
Q

What is the effect of damage Meyer’s loop carrying information from the inferior retina and thus the SUPERIOR VISUAL FIELD?

A

resulting in loss of vision in the superior nasal field of the left eye and the superior temporal field of the right eye

How well did you know this?
1
Not at all
2
3
4
5
Perfectly
15
Q

What is the effect of Damage to the left Baum’s loop carrying information from the superior retina and thus the INFERIOR VISUAL FIELD?

A

resulting in loss of vision in the inferior temporal field of the right eye and the inferior nasal field of the left eye

How well did you know this?
1
Not at all
2
3
4
5
Perfectly
16
Q

What are the 7 extraocular muscles, what are their movements, and what are their innervations?

A

Levator Palpabrae Superiosis- Oculomotor nerve, raises the eyelid
Superior Rectus- Oculomotor nerve, lifts the eyeball superiorly and posteriorly
Lateral rectus- Abducens Nerve, Abducts the eyeball
Inferior rectus- Oculomotor nerve, pulls the eyeball inferiorly and posteriorly
Medial Rectus- Oculomotor nerve, adducts the eyeball
Superior Oblique- Trochlear nerve, External rotation is known as extortion (AWAY from MIDLINE)
Inferior Oblique- Oculomotor nerve, Internal rotation is known as intortion (TOWARDS MIDLINE)

How well did you know this?
1
Not at all
2
3
4
5
Perfectly
17
Q

Define Anterograde?

A

Transport from neuronal cell bodies to axon terminals

How well did you know this?
1
Not at all
2
3
4
5
Perfectly
18
Q

Define retrograde?

A

Transport from axonal terminals to neuronal cell bodies

Neurones do not project from A —> B instead its from A—> B —> C —> D etc.

How well did you know this?
1
Not at all
2
3
4
5
Perfectly
19
Q

Define a Class A experiment?

A

DIAGNOSIS: - Some behavioural, physiological or pharmacological variable is manipulated and the consequent effects on brain activity/structure are measured i.e. DIAGNOSIS

How well did you know this?
1
Not at all
2
3
4
5
Perfectly
20
Q

Define a Class B experiment?

A

TREATMENT: - Some aspect of the brain structure (lesion) or activity (stimulation/inhibition) is manipulated and the consequent effects on behaviour/physiology/ endocrinology is measured i.e. TREATMENT

How well did you know this?
1
Not at all
2
3
4
5
Perfectly
21
Q

Define Gastrulation?

A

single layer blastula developing into tri-laminar disc (gastrula)
The embryo develops into a tri-laminar disc made up of:
- Ectoderm
- Mesoderm
- Endoderm

How well did you know this?
1
Not at all
2
3
4
5
Perfectly
22
Q

Define neuralation?

A

the process of formation of the embryonic nervous system

How well did you know this?
1
Not at all
2
3
4
5
Perfectly
23
Q

Describe the formation of the neural tube?

A

1 The ECTODERM thickens in the mid-line to form the neural plate in the 3rd week of embryonic development
2 - The ectoderm then undergoes differential mitosis to cause the formation of a mid-line groove known as the neural groove
3 - This groove deepens and eventually detaches from the overlying ectoderm to form the neural tube
4 - Lateral to the neural plate lie presumptive neural crest cells which run dorso-laterally along the neural groove
5 - Neural crest cells develop to form many cell types:
• Sensory (dorsal root) ganglia of the spinal cord and cranial nerves V,VII, IX & X
• Schwann cells
• Pigment cells
• Adrenal medulla
• Bony skull
• Meninges
Dermis
• [Quite a lot of the head & neck are made up of neural crest cells]
6 - During embryonic development the rostral (i.e. superior) portion of the neural tube, which develops into the brain (CNS), grows faster than the caudal (i.e.inferior) portion, which develops into the spinal cord (the central cavity within the neural tube becomes the central canal of the spinal cord and the ventricles of the brain)
7 - By the 5th week of embryonic development, three primary brain vesicles can be identified: 1. Prosencephalon (forebrain) 2. Mesencephalon (midbrain) 3. Rhombencephalon (hindbrain)
8 - By the 7th week, further differentiation occurs resulting in the formation of secondary brain vesicles: 1. Prosencephalon —> Telencephalon & Diencephalon 2. Mesencephalon —> Mesencephalon 3. Rhombencephalon —> Metencephalon & Myelencephalon
9 - The secondary brain vesicles give rise to derivatives in the mature brain: • Telencephalon —> Cerebral hemisphere & Lateral ventricles • Diencephalon —> Thalamus, Hypothalamus & Third ventricle • Mesencephalon —> Midbrain (colliculi) & Aqueduct • Metencephalon —> Cerebellum, Pons & Upper part of fourth ventricle • Myelencephalon —> Medulla oblongata & Lower part of fourth ventricle
10 - As the brain develops, its central cavity also undergoes considerable changes in size and shape, forming a system of chambers called ventricles with contain cerebrospinal fluid

How well did you know this?
1
Not at all
2
3
4
5
Perfectly
24
Q

Describe abnormalities in the spinal cord development?

A

The neural tube usually closes at the end of the 4th week
Failure of the tube to close in the spinal cord results in - spina bifida
Failure of the tube to close in the cephalic region (brain) results in - anencephalus
The reasons for failure to close could be due to faulty induction or environmental teratogens (any agent that can disturb the development of the embryo) acting on neuroepithelial cells

How well did you know this?
1
Not at all
2
3
4
5
Perfectly
25
Q

What are the development milestones in neural development?

A
By 3 weeks there is eye formation
 10 weeks = cerebral expansion & commissures
 3 months = basic structures established
5 months = myelination has begun
7 months = lobes cerebrum has formed 
 9 months = gyri and sulci formed
How well did you know this?
1
Not at all
2
3
4
5
Perfectly
26
Q

What are the critical periods of neural development?

A

Abnormalities to the CNS are dependent on time of infection
6th week - eye malformations occur e.g cataracts
9th week - deafness can occur e.g. malformation of the organ of corti
5th to 10th week - cardiac malformation occur
In general CNS disorders generally occur in the 2nd trimester
The risk of disorders falls after 16 weeks due to the fact that most of the structures of the CNS have developed by this time

How well did you know this?
1
Not at all
2
3
4
5
Perfectly
27
Q

Describe the development of sensation?

A

There is innervation of dermal skin from 28 weeks
The dorsal root ganglion connects to the spinal cord from 8 weeks but this is nonnoxious (i.e. no pain is detected)
C-fibre connection (noxious (painful) stimuli) from 19+ weeks
Organised thalamus from 8+ weeks
Retinal inputs arrive at 14-16 weeks
Myelination occurs from 25 weeks (speed of conduction increases with myelination)
Connections from the thalamus to the cortex occur from 24 weeks

How well did you know this?
1
Not at all
2
3
4
5
Perfectly
28
Q

Describe the features of the blood brain barrier?

A

Endothelial tight junctions
Astrocyte end feet
Pericytes
Continuous basement membrane, lacks fenestrations (windows)
Requires specific transported for glucose, essential ions etc.
Certain parts of the brain lack the blood-brain barrier, these are called CIRCUMVENTRULAR ORGANS e.g. posterior pituitary - they need to be in contact with the blood for a sensory role to monitor

How well did you know this?
1
Not at all
2
3
4
5
Perfectly
29
Q

Describe CSF and its transport?

A

Circulates through the subarachnoid space (around the brain and spinal cord) and within ventricles - offers protection by cushioning brain from gentle movements)

There are four ventricles: 1. Lateral (paired) 2. III 3. IV • Ventricles & subarachnoid spaces connect via cisterns

Entries CSF is around 120mls

Its a clear, colourless liquid which contains; protein, urea, glucose & salts

Produced by ependymal cells in the choroid plexuses of the lateral ventricles (mainly)

Choroid plexus: - Formed from modified ependymal cells - They from around a network of capillaries, large surface area

Absorbed via arachnoid granulations (VILLI) e.g. in the superior sagittal sinus

How well did you know this?
1
Not at all
2
3
4
5
Perfectly
30
Q

What is hydrocephalus?

A

Abnormal accumulation of CSF in ventricular system. Often due to a blocked cerebral aqueduct - Accumulation of fluid leads to a build up of pressure which can damage brain tissue since the skull in hard in adults - In children with soft skull the pressure will cause the soul to bulge and look abnormal as well as damaging the brain

How well did you know this?
1
Not at all
2
3
4
5
Perfectly
31
Q

What is gastrulation?

A

single layer blastula developing into tri-laminar disc (gastrula)
The embryo develops into a tri-laminar disc made up of:
- Ectoderm
- Mesoderm
- Endoderm

How well did you know this?
1
Not at all
2
3
4
5
Perfectly
32
Q

What is neurultion?

A

the process of formation of the embryonic nervous system

How well did you know this?
1
Not at all
2
3
4
5
Perfectly
33
Q

Describe the formation of the neural tube?

A

1 The ECTODERM thickens in the mid-line to form the neural plate in the 3rd week of embryonic development
2 - The ectoderm then undergoes differential mitosis to cause the formation of a mid-line groove known as the neural groove
3 - This groove deepens and eventually detaches from the overlying ectoderm to form the neural tube
4 - Lateral to the neural plate lie presumptive neural crest cells which run dorso-laterally along the neural groove
5 - Neural crest cells develop to form many cell types:
• Sensory (dorsal root) ganglia of the spinal cord and cranial nerves V,VII, IX & X
• Schwann cells
• Pigment cells
• Adrenal medulla
• Bony skull
• Meninges
Dermis
• [Quite a lot of the head & neck are made up of neural crest cells]
6 - During embryonic development the rostral (i.e. superior) portion of the neural tube, which develops into the brain (CNS), grows faster than the caudal (i.e.inferior) portion, which develops into the spinal cord (the central cavity within the neural tube becomes the central canal of the spinal cord and the ventricles of the brain)
7 - By the 5th week of embryonic development, three primary brain vesicles can be identified: 1. Prosencephalon (forebrain) 2. Mesencephalon (midbrain) 3. Rhombencephalon (hindbrain)
8 - By the 7th week, further differentiation occurs resulting in the formation of secondary brain vesicles: 1. Prosencephalon —> Telencephalon & Diencephalon 2. Mesencephalon —> Mesencephalon 3. Rhombencephalon —> Metencephalon & Myelencephalon
9 - The secondary brain vesicles give rise to derivatives in the mature brain: • Telencephalon —> Cerebral hemisphere & Lateral ventricles • Diencephalon —> Thalamus, Hypothalamus & Third ventricle • Mesencephalon —> Midbrain (colliculi) & Aqueduct • Metencephalon —> Cerebellum, Pons & Upper part of fourth ventricle • Myelencephalon —> Medulla oblongata & Lower part of fourth ventricle
10 - As the brain develops, its central cavity also undergoes considerable changes in size and shape, forming a system of chambers called ventricles with contain cerebrospinal fluid

How well did you know this?
1
Not at all
2
3
4
5
Perfectly
34
Q

Describe abnormalities in the spinal cord?

A

The neural tube usually closes at the end of the 4th week
Failure of the tube to close in the spinal cord results in - spina bifida
Failure of the tube to close in the cephalic region (brain) results in - anencephalus
The reasons for failure to close could be due to faulty induction or environmental teratogens (any agent that can disturb the development of the embryo) acting on neuroepithelial cells

How well did you know this?
1
Not at all
2
3
4
5
Perfectly
35
Q

What are the developmental milestones?

A
By 3 weeks there is eye formation
 10 weeks = cerebral expansion & commissures
 3 months = basic structures established
5 months = myelination has begun
7 months = lobes cerebrum has formed 
 9 months = gyri and sulci formed
How well did you know this?
1
Not at all
2
3
4
5
Perfectly
36
Q

What are the critical period of neural development?

A

Abnormalities to the CNS are dependent on time of infection
6th week - eye malformations occur e.g cataracts
9th week - deafness can occur e.g. malformation of the organ of corti
5th to 10th week - cardiac malformation occur
In general CNS disorders generally occur in the 2nd trimester
The risk of disorders falls after 16 weeks due to the fact that most of the structures of the CNS have developed by this time

How well did you know this?
1
Not at all
2
3
4
5
Perfectly
37
Q

Describe the development of sensation?

A

There is innervation of dermal skin from 28 weeks
The dorsal root ganglion connects to the spinal cord from 8 weeks but this is nonnoxious (i.e. no pain is detected)
C-fibre connection (noxious (painful) stimuli) from 19+ weeks
Organised thalamus from 8+ weeks
Retinal inputs arrive at 14-16 weeks
Myelination occurs from 25 weeks (speed of conduction increases with myelination)
Connections from the thalamus to the cortex occur from 24 weeks

How well did you know this?
1
Not at all
2
3
4
5
Perfectly
38
Q

What is a fasciculi?

A

Nerve axons run up and down the spinal cord in bundles called fasiculi
The axons in a fasiculus are from neurones that come from or are going to similar locations

How well did you know this?
1
Not at all
2
3
4
5
Perfectly
39
Q

Where does the dorsal/medial leminiscal column decussate?

A

In the medulla

How well did you know this?
1
Not at all
2
3
4
5
Perfectly
40
Q

What sensation is carried by the dorsal/medial leminiscal column?

A

Proprioception, vibration, and descriminative touch

How well did you know this?
1
Not at all
2
3
4
5
Perfectly
41
Q

Describe the pathway of the dorsal/medial leminiscal column?

A

Fasciculus cuneatus - LATERAL and carries information from the UPPER body to the cuneate tubercle in the medulla

Fasciculus gracilis - MEDIAL and carries information from the LOWER body to the gracile tubercle in the medulla

Ascends to the medulla and then DECUSSATES to become the medial lemniscus then ascends to the thalamus then to the cortex

transmits to the pre-central gyrus - primary somatosensory cortex
• There are 4 sensory nerve endings that may sense fine touch (in the skin) (beginning the pathway):
1. Meissner’s corpuscle
2. Pacinian corpuscle
3. Ruffini endings
4. Merkel endings

• There are three neurones in this pathway:

  1. in dorsal root ganglion
  2. in the cuneate & gracile nuclei
  3. in the ventral posterolateral nucleus of the thalamus
How well did you know this?
1
Not at all
2
3
4
5
Perfectly
42
Q

What are the ascending tracts?

A

Dorsal/medial leminiscal column, spinothalamic tract, spinocerebellar tract, spinoreticular tract

How well did you know this?
1
Not at all
2
3
4
5
Perfectly
43
Q

Where does the spinothalamic tract decussate?

A

Across the white anterior commissure in the spinal cord

How well did you know this?
1
Not at all
2
3
4
5
Perfectly
44
Q

What sensation is carried by the spinothalamic tract?

A

LATERAL: pain & temperature

MEDIAL: crude touch

How well did you know this?
1
Not at all
2
3
4
5
Perfectly
45
Q

Describe the pathway of the spinothalamic tract?

A

Enters the spinal cord at lassauer’s fasciculus
Ascends 1-2 segments ipsilaterally then synapses with the cell bodies of the substantia gelatinosa (dorsal horn) - first order neurone
The secondary afferent decussates immediately across the white anterior commissure - second order neurone
The two tracts (lateral and medial) ascend and eventually join at the medulla to form the spinal lemniscus and ends at the thalamus - second order neurone

• Tract travels from the dorsal horn to the CONTRALATERAL thalamus
• The tract terminates at the thalamus
- From the thalamus the tertiary afferent goes from the thalamus to the somatosensory cortex - third order neurone

• Three neurones pathway:

  1. in the dorsal root ganglion
  2. in the dorsal horn of the spinal cord
  3. in the thalamus
How well did you know this?
1
Not at all
2
3
4
5
Perfectly
46
Q

What sensation is carried by the spinocerebellar tract?

A

Ventral spinocerebellar tract: carries information on proprioception to the ventral CONTRA & IPSILATERAL cerebellum

Dorsal spinocerebellar tract: carries information on proprioception to the dorsal IPSILATERAL cerebellum

How well did you know this?
1
Not at all
2
3
4
5
Perfectly
47
Q

What sensation is carried by the spinoreticular tract?

A

Carries deep/chronic pain

How well did you know this?
1
Not at all
2
3
4
5
Perfectly
48
Q

What is an overview of the descending tracts?

A

Originate from the cerebral cortex and brainstem (UPPER MOTOR NEURONES)

Their role concerns the control of movement, muscle tone, spinal reflexes, spinal autonomic functions & transmission of sensory information to higher centres

Divided into pyramidal & extrapyramidal

There are no synapses within the descending pathways. At the termination of the descending tracts, the neurones synapse with a lower motor neurone. Thus, all the neurones within the descending motor system are classed asupper motor neurones. Their cell bodies are found in the cerebral cortex or the brain stem, with their axons remaining within the CNS.

How well did you know this?
1
Not at all
2
3
4
5
Perfectly
49
Q

What are the divisions of the descending tracts and explain them?

A

Pyramidal-• 2 neurone pathway originating in the cerebral cortex of cranial nerve nucleus (for facial innervation)
DECUSSATE in the medulla and descend CONTRALATERALLY
• Neurones innervating our axial muscles (muscles of the trunk and head) mostly do not decussate
• Synapse with the cell bodies of the ventral horn of the spinal grey matter

Extra-pyramidal- • Originate in the brainstem and carry motor fibres to the spinal cord
• Responsible for involuntary autonomic control of all musculature
• Decussating tracts - contralateral innervation:

How well did you know this?
1
Not at all
2
3
4
5
Perfectly
50
Q

Which descending tracts are pyramidal?

A

Corticospinal tract and the Corticobulbar tract

How well did you know this?
1
Not at all
2
3
4
5
Perfectly
51
Q

Which descending tracts are extra-pyramidal?

A

Tectospinal tract, Rubrospinal tract, Reticulospinal tract, vestibulospinal tract

How well did you know this?
1
Not at all
2
3
4
5
Perfectly
52
Q

Where does the corticospinal tract decussate?

A
  • Lateral (75%): pyramidal (medulla) decussation - limb muscles
  • Medial (25%): decussates as it leaves via the anterior white commissure (a bundle of nerve fibres that cross the mid-line of the spinal cord) - axial muscles (head & trunk)
How well did you know this?
1
Not at all
2
3
4
5
Perfectly
53
Q

What innervation does the corticospinal tract provide?

A

Transmits control of voluntary muscles (motor)

How well did you know this?
1
Not at all
2
3
4
5
Perfectly
54
Q

What is the pathway of the corticospinal tract?

A

• Begins in the pre-central gyrus - primary motor cortex
receivea range of inputs:
Primary motor cortex
Premotor cortex
Supplementary motor area
• Neurones ultimately innervate the axial and limb muscles - they leave the cortex by descending through the internal capsule and into the brainstem
• Upper motor neurones (UMN) originate in the motor cortex - a UMN lesion can occur anywhere from the cortex all the way down to the ventral horn
• Neurones (cell bodies) located in the ventral horns project to limb and axial muscles - these are the lower motor neurones

• Two neurones involved:

  1. Upper motor neurones:
    - Limb efferents decussate at medullary pyramids
    - Axial efferents decussates at appropriate levels

Theanterior corticospinal tractremains ipsilateral, descending into the spinal cord. They then decussate and terminate in the ventral horn of thecervicalandupperthoracicsegmental levels.

How well did you know this?
1
Not at all
2
3
4
5
Perfectly
55
Q

What is carried by the corticobulbar tract?

A

• Innervation of the skeletal muscles of the head and neck via the cranial nerves

How well did you know this?
1
Not at all
2
3
4
5
Perfectly
56
Q

Describe the corticobulbar tract?

A

It has no decussation
• Bilateral innervation
• EXCEPT for cranial nerves; facial (VII), hypoglossal (XII) & glossopharyngeal (IX) which have contralateral innervation
The corticobulbar tracts arise from the lateral aspect of theprimary motor cortex. They receive the same inputs as the corticospinal tracts. The fibres converge and pass through the internal capsule to thebrainstem.
The neurones terminate on the motor nuclei of thecranial nerves.Here, they synapse with lower motor neurones, which carry the motor signals to the muscles of thefaceandneck.
Clinically, it is important to understand the organisation of the corticobulbar fibres. Many of these fibres innervate the motor neuronesbilaterally. For example, fibres from the left primary motor cortex act as upper motor neurones for the right and left trochlear nerves. There are a few exceptions to this rule:
Upper motor neurones for thefacial nerve(CN VII) have a contralateral innervation.
Upper motor neurons for thehypoglossal(CN XII) nerve only providecontralateralinnervation.

How well did you know this?
1
Not at all
2
3
4
5
Perfectly
57
Q

What is the function of the tectospinal tract?

A

The tectospinal tract coordinates movements of the head in relation tovision stimuli.

How well did you know this?
1
Not at all
2
3
4
5
Perfectly
58
Q

What is the pathway of the tectospinal tract and where does it decussate?

A

Arises from the tectum; inferior (auditory) & superior (visual) colliculus
This pathway begins at thesuperior colliculusof the midbrain. The superior colliculus is a structure that receives input from theoptic nerves.The neurones then quickly decussate, and enter the spinal cord. They terminate at the cervical levels of the spinal cord.

How well did you know this?
1
Not at all
2
3
4
5
Perfectly
59
Q

What is the function of the rubrospinal tract?

A
  • Assists in motor functions

* Fine hand movements

How well did you know this?
1
Not at all
2
3
4
5
Perfectly
60
Q

What is the pathway of the rubrospinal tract and where does it decussate?

A

The rubrospinal tract originates from thered nucleus, a midbrain structure. As the fibres emerge, they decussate (cross over to the other side of the CNS), and descend into the spinal cord. Thus, they have acontralateralinnervation.

How well did you know this?
1
Not at all
2
3
4
5
Perfectly
61
Q

What is the function of the vestibulospinal tract?

A
  • Muscle tone

* Balance & posture by innervating the antigravity muscles

How well did you know this?
1
Not at all
2
3
4
5
Perfectly
62
Q

What is the pathway of the vestibulospinal tract and where does it decussate?

A

no decussation
• From the vestibular nuclei
There are two vestibulospinal pathways; medial and lateral. They arise from thevestibular nuclei, which receive input from the organs of balance. The tracts convey this balance information to the spinal cord, where it remainsipsilateral.
Fibres in this pathway controlbalanceandpostureby innervating the ‘anti-gravity’ muscles (flexors of the arm, and extensors of the leg), via lower motor neurones.
Lateral:
Fibres descend ipsilaterally though the anterior funiculus of the same side of the spinal cord, synapsing on the extensor antigravity motor neurons
Medial:
Descends bilaterally in the medial longitudinal fasciculus
Synapses with the excitatory and inhibitory neurons of the cervical spine

How well did you know this?
1
Not at all
2
3
4
5
Perfectly
63
Q

What is the role of the reticulospinal tract?

A

Responsible for spinal reflexes
The two recticulospinal tracts have differing functions:
Themedial reticulospinal tractarises from thepons. It facilitates voluntary movements, and increases muscle tone.
Thelateral reticulospinal tractarises from themedulla. It inhibits voluntary movements, and reduces muscle tone

How well did you know this?
1
Not at all
2
3
4
5
Perfectly
64
Q

What is the pathway of the reticulospinal tract and where does it decussate?

A
  • Medial pathway: Mediated by the pons which controls the extensors acting to increase muscle tone thereby facilitating voluntary movement
  • Lateral pathway: Mediated by the medulla which controls the flexors acting to decrease muscle tone thereby inhibiting voluntary movement

No decussation

How well did you know this?
1
Not at all
2
3
4
5
Perfectly
65
Q

Explain Brown-Sequard Syndrome and what would be experienced?

A

It is key to think about both ascending and descending tracts:

• Damage to a hemi-section of the spinal cord
• Ipsilateral & contralateral are in relation TO THE LESION
• Ipsilateral weakness (i.e less motor etc.) below the lesion - due to damage to the ipsilateral descending motor corticospinal tract (decussated at the medulla already)
• Ipsilateral loss of dorsal column proprioception below lesion - since the ascending tracts are damaged before they could decussate in the medulla
Contralateral loss of spinothalamic pain & temperature below the lesion since spinothalamic fibres decussate just after entering cord within the spinal cord

• Overall: - Ipsilateral loss of; proprioception, motor & fine touch - Contralateral loss of; pain, temperature & crude touch

How well did you know this?
1
Not at all
2
3
4
5
Perfectly
66
Q

What is the human hearing range and at what range are our ears most sensitive?

A

Hearing range is from 20 to 20,000 Hz

The ear is most sensitive at 1000 - 4000Hz

How well did you know this?
1
Not at all
2
3
4
5
Perfectly
67
Q

What are the three segments of the ear and what is a brief overview of their roles?

A

Outer Ear: helps you collect sound

Middle Ear: transmission of sound

Inner Ear: conversion of sound into neural impulses

How well did you know this?
1
Not at all
2
3
4
5
Perfectly
68
Q

Describe the process of sound transmission in the external ear?

A
  • Sound first enters the ear through the pinna (or auricle) which is the exterior part of the ear
  • It then enters the ear via the external auditory canal/meatus
  • The shape of both the pinna (or auricle) & external auditory canal/meatus help to amplify and direct the sound
  • The sound then makes its way through the canal, to the tympanic membrane (eardrum)
  • As the air molecules push against the membrane, it causes the tympanic membrane to vibrate at the same frequency as the sound wave
  • The membrane vibrates slowly to low frequency sounds and very rapidly to high frequency sounds
  • The tympanic membrane marks the end of the external ear and marks the start of the middle ear
How well did you know this?
1
Not at all
2
3
4
5
Perfectly
69
Q

What is the middle ear, which nerve provides sensation, and what is the role of eustachian tube in the ear?

A

• An air-filled cavity in the temporal bone of the skull
• Sensation of the middle ear is provided by the glossopharyngeal nerve CNIX
The pressures in the external auditory canal/meatus and middle ear cavity are normally equal to atmospheric pressure
• The middle ear is exposed to atmospheric pressure via the eustachian tube (or auditory tube) which connects the middle ear to the pharynx
• The eustachian tube opens into the pharynx through a slit-like opening which is normally closed, EXCEPT when muscle movements result in the opening of the tube during swallowing, yawning or sneezing
• A difference in pressure between the middle and external ear occurs due to changes in altitude
• When the pressure outside the ear and in the external auditory meatus change, the middle ear initially remains constant due to the fact that the eustachian tube is closed
• This constant pressure can stretch the tympanic membrane resulting in pain - which can be relieved by yawning/swallowing which in turn results in the opening of the eustachian tube thereby allowing the pressure in the middle ear to equilibrate with the external atmospheric pressure

How well did you know this?
1
Not at all
2
3
4
5
Perfectly
70
Q

What is the process of sound transmission in the middle ear?

A
  • The vibrations of the tympanic membrane are transmitted to the inner ear (for processing) through a moveable chain of three bones - the ossicles (the smallest bones in the body)
  • Vibrations from the tympanic membrane are transmitted into the inner via firstly through the malleus then incus then stapes (MIS)
  • These bones have synovial joints between them
  • They act as a piston and couple the vibrations of the tympanic membrane to the OVAL WINDOW (a membrane covered opening between the middle and inner ear)
  • The total force of a sound wave applied to the tympanic membrane is completely transferred to the oval window
  • However, due to the fact that the oval window is much smaller than the tympanic membrane, the force per area is much greater which is required to adequately transmit the sound energy through the FLUID FILLED COCHLEA
How well did you know this?
1
Not at all
2
3
4
5
Perfectly
71
Q

How is the amount of energy transferred to the inner ear controlled?

A

The amount of energy transmitted to the inner ear can be lessened by the contraction of two small muscles in the middle ear - the tensor tympani (innervated by V3 (mandibular branch trigeminal) & stapedius (innervated by CN7 (facial))
• The tensor tympani attaches to the malleus, and contraction of the muscle dampens the bones movement - innervated by the mandibular division (V3) of the TRIGEMINAL NERVE CN5
• The stapedius attaches to the stapes and similarly controls it - innervated by the FACIAL NERVE CN7
• These muscles act reflexively to CONTINUOUS LOUND NOISE to protect the delicate receptor apparatus in the inner ear
• These muscles CANNOT protect the inner ear from SUDDEN INTERMITTENT LOUND SOUNDS

How well did you know this?
1
Not at all
2
3
4
5
Perfectly
72
Q

Describe the anatomy of the inner ear?

A
  • The inner ear is called the cochlea (the organ of hearing)
  • The cochlea is a spiral-shaped (coiled around 2.5 - 2.75 times), fluid filled space in the temporal bone
  • The cochlea is almost completely divided lengthwise by a membranous tube called the cochlear duct - which contains the sensory receptors of the auditory system
  • The cochlea duct is filled with a fluid called endolymph - a compartment of extracellular fluid containing a high concentration of K+ and a low concentration of Na+ (this arrangement of concentrations is normally seen in intracellular fluid)
  • On either side of the cochlear duct are compartments filled with perilymph, which is similar in composition to cerebrospinal fluid (CSF)
  • The scala vestibuli is above the cochlear duct and begins at the oval window - remember it since vestibule is like an entrance so forms the entrance to the inner ear from the oval window
  • The scale tympani is below the cochlear duct and connects to the middle ear via a second-membrane covered opening, the round window
  • The scala vestibuli and tympani are continuous at the far end of the cochlear duct at the helicotrema
How well did you know this?
1
Not at all
2
3
4
5
Perfectly
73
Q

What is the process of sound transmission in the inner ear?

A
  • Sounds waves from the external acoustic meatus cause the tympanic membrane to move in and out which in turn is transmitted to the ossicles which in turn transmit this movement to the oval window
  • This results in the oval window moving in and out of the scala vestibuli
  • This movement creates waves of pressure in the scala vestibuli
  • The majority of these waves of pressure are transmitted across the cochlear duct with some being transmitted toward the helicotrema and into the scala tympani where the pressure is relieved by the movements of the membrane of the round window
  • The side of the cochlear duct closest to the scala tympani is formed by the basilar membrane, upon which sits the ORGAN OF CORTI - contains the ears sensitive receptor cells, pressure difference across the duct causes the basilar membrane to vibrate
  • The base of the basilar membrane is narrow & stiff and thus sensitive to high frequencies
  • The apex of the basilar membrane is wider & less stiff and thus is sensitive to low frequencies
How well did you know this?
1
Not at all
2
3
4
5
Perfectly
74
Q

Explain the hair cells of the organ of corti?

A
  • These cells are mechanoreceptors that have hairlike stereo-cilia protruding from one end
  • Some antibiotics can damage the stereocilia of the hair cells
  • There are two anatomically separate groups of hair cells; a SINGLE ROW of inner hair cells & 4-5 ROWS of outer hair cells
  • The stereocilia of inner hair cells extend into the endolymph fluid and convert pressure waves caused by the movement of fluid in the cochlear duct into receptor potentials
  • The stereocilia of the outer hair cells are embedded in the overlying tectorial membrane (see diagram above) and mechanically alter its movement to sharpen frequency tuning at each point along the basilar membrane
  • The tectorial membrane overlies the organ of corti, as the pressure waves displace the basilar membrane, the hair cells move in relation to the stationary tectorial membrane resulting in the bending of the stereocilia
  • When the stereocilia bend towards the tallest member of the bundle, fibrous connections called TIP LINKS pull open mechanically gated K+ channels, resulting in an influx of K+ from the surrounding endolymph (K+ rich) thereby depolarising the membranes
  • This change in voltage triggers the opening of voltage-gated Ca2+ channels near the base of the cell, which in turn triggers neurotransmitter release (since Ca2+ causing neurotransmitter containing vesicles to migrate to the presynaptic membrane)
  • Bending the hair cells in the opposite direction, slackens the tip links thereby closing the channels and allowing the cell to rapidly repolarize
  • The neurotransmitter released from the hair cells is GLUTAMATE which in turn binds to and activates proteinbinding sites on the terminals of the afferent neurones
  • As sound waves vibrate the basilar membrane, the stereocilia are bent back and forth, the membrane potential of the hair cells rapidly oscillates and bursts of glutamate are released onto afferent neurones
  • This results in the generation of action potentials in the neurons, the axons of which join to form the COCHLEAR BRANCH of the VESTIBULOCOCHLEAR NERVE (CN VIII)
  • The greater the energy (loudness) of the sound wave, the greater the frequency of action potentials generated in the afferent nerve fibres
  • Due to its position on the basilar membrane, each hair cell respond to a limited range of sound frequencies, with one particular frequency stimulating it most strongly
How well did you know this?
1
Not at all
2
3
4
5
Perfectly
75
Q

Explain the neural pathway of hearing in the Cochlear branch?

A

• Cochlear nerve fibres make dendritic contact with hair cells of the organ of corti within the cochlear duct - the cell bodies of these fibres lie within the cochlea and are collectively called the spiral ganglion
• The cochlear nerve joins the brainstem at the level of the rostral medulla
• Its fibres bifurcate and end in the dorsal & ventral cochlear nuclei, which lie close to the inferior cerebellar peduncle
• From the cochlear nuclei, the second-order neurones ascend into the pons where the fibres travel to the superior olivary nucleus
• The superior olivary nucleus has fibres that leaves the brainstem in the vestibulocochlear nerve and end in the organ of corti, these serve an inhibitory function and serve to adjust transmission of auditory information through the cochlear nerve by mediating contractions of the tensor tympani and stapedius in response to loud noises
• From the superior olivary nucleus the fibres travel to the inferior colliculus of the midbrain
• From here the inferior brachium (nerve fibre) carries the auditory information to the medial geniculate body of the thalamus
• From the medial geniculate body, fibres travel through the internal capsule to the primary auditory cortex of the temporal lobe - situated on the dorsal surface of the superior temporal gyrus
• The region of the temporal lobe surrounding the primary auditory cortex is called Wernicke’s area (auditory association cortex) and it is here where auditory information is interpreted and understood - it is located in the superior temporal lobe. Wernicke’s area is important in the processing of language in the brain. So if Wernicke’s area is damaged then a patient will not be able to understand questions and their speech will be incomprehensible
Summary of path of cochlear branch of vestibulocochlear nerve (CNVIII): - Cochlear nuclei - Superior olivary nucleus - Inferior colliculus - Medial geniculate body - via the inferior brachium (nerve) - Primary auditory cortex, in Wernicke’s area
To remember that medial geniculate body use: I’MAuditory - Inferior colliculus, Medial geniculate body, for Auditory information • Thus by default, the superior colliculus & lateral geniculate body must be for vision
NOTE: the chorda tympani (branch of the facial nerve CNVII) conveys taste information from the tongue and runs THROUGH THE MIDDLE EAR to carries taste messages to the brain • Bell’s palsy: Acute unilateral inflammation of the facial nerve, results in pain behind the ear (due to the chorda tympani and also the facial nerve in the interior auditory meatus), paralysis of facial muscles & failure to close eye
the facial nerve CNVII exits the cranial cavity into the internal acoustic meatus behind the cochlea, thus any inflammation of the nerve may result in pain behind the ear (e.g. in Bell’s palsy)

76
Q

What is the overall mechanism of the vestibular system?

A
  • Hair cells can also be found in the vestibular apparatus of the inner ear
  • The vestibular apparatus is a connected series of ENDOLYMPH-FILLED, membranous tubes that also connect with the cochlear duct
  • The hair cells detect changes in the motion and position of the head by a stereocilia transaction mechanism
  • The vestibule apparatus consists of THREE membranous SEMICIRCULAR CANLAS & two saclike swellings; the UTRICLE & SACCULE - all of which lie in the temporal bone on either side of the head
77
Q

Describe in detail the role of the semi-circular canals in the vestibular system?

A
  • Detect angular acceleration during the ROTATION of the head along three perpendicular axes
  • activated when nodding the head up and down, shaking the head from side to side & tipping the head so that the ear touches the shoulder
  • The receptor cells of the semicircular canals, like the organ of corti, contain stereocilia
  • These stereocilia are encapsulated within a gelatinous mass, the CAPULA which extends across the lumen of each semicircular canal at the ampulla (a slight bulge in the wall of each duct)
  • Whenever the head moves, the semicircular canal within its bony enclosure and the attached bodies of the hair cells all move with it
  • The fluid (endolymph) filling the duct however, is not attached to the skull, and due to inertia, remains in its original position
  • Thus, the moving ampulla is pushed against the stationary fluid which in turn causes bending of the stereocilia and the alteration in the rate of release of glutamate from the hair cells
  • Glutamate then crosses the synapse and activates neurones associated with the hair cells, initiating the propagation of action potential towards the brain
  • The speed & magnitude of rotational head movements determine the direction in which the stereocilia are bent and which hair cells are stimulated
  • Glutamate is released from the hair cells at rest and the release increases or decrease from the resting rate according to the direction in which the hairs are bent
  • Each hair cell receptor has one direction of maximum neurotransmitter release so when its stereocilia are bent in this direction, the receptor cell depolarises. When the stereocilia are bent in the opposite direction the cell hyperpolarises
  • When the head continuously rotates at a steady velocity the duct fluid begins to move at the same rate as the rest of the head and the stereocilia slowly return to resting position
  • thus hair cells are only stimulated during acceleration or deceleration
  • Damage to the canals of one side result in nystagmus (rapid, jerky, back and fourth movement of the eyes) with the slow phase towards the damaged side and the rapid reset away from the damaged side
  • Pouring ice cold water into the external auditory meatus can cause convection currents in the semicircular calls resulting in nystagmus
78
Q

Describe the role of the ultricle and saccule in the vestibular system?

A

Provide information about linear acceleration of the head, and about the changes in head position in relation to gravity
The hair cells in the utricle point nearly straight up when standing and they respond when the head is tipped away from the horizontal plane or to linear accelerations in the horizontal plane
- The hair cells of the saccule project at right angles to those of the utricle and they responded when you move from lying to a standing position or to vertical accelerations like those produced when jumping on a trampoline
- The stereocilia projecting from the hair cells are covered by a gelatinous substance in which tiny stones known as otoliths are embedded
- The stereocilia project into the otolithic membrane
- The otoliths with are calcium carbonate crystals make the gelatinous substance heavier than the surrounding endolymph
- In response to a change in position the gelatinous otolithic material moves according to the forces of gravity and pulls against the hair cells so that the stereocilia on the hair cells bend and the receptor cells are stimulated and the action potential is propagated via the VESTIBULAR NERVE (branch of the vestibulocochlear nerve (CNVIII))

79
Q

How is vestibular information used?

A
  1. Control of eye muscles so that in spite of changes to head position, the eyes can remain fixed on the same point. Nystagmus is a large,jerky, back-and-fourth movement of the eyes that can occur in response to unusual vestibular input in healthy people, but can also be used as a sign of pathology
  2. Reflex mechanisms of maintaining upright posture & balance. The vestibular apparatus plays a role in the support of the head during movement and orientation of the head in space
  3. Conscious awareness of the position & acceleration of the body, perception of the space surrounding the body and memory of spatial information - i.e PROPRIOCEPTION
80
Q

What is the neural vestibular pathway?

A

The central processes of vestibular fibres mostly end in the vestibular nuclei of the rostral medulla

81
Q

What is an acoustic neuroma and what are the symptoms?

A

is a benign tumour of myelin sheath of CN VIII (vestibulocochlear).

(unilateral) deafness, dizziness, fullness in the ear and tinnitus. If undetected it progresses to include ataxia and paralysis of cranial nerves VII and V, this is termed cerebellopontine angle syndrome. Around 5%-10% are caused by rare genetic condition neurofibromatosis type 2

82
Q

What is jugular foramen syndrome and what are the symptoms?

A

Compression of multiple lower cranial nerves (IX,X XI)

leads to signs and symptoms including dysphonia, loss of gag reflex and unilateral wasting of sternocleidomastoid and trapezius muscles.

83
Q

What is foramen magnum syndrome and what are the symptoms?

A

compression of the spinal cord, lower brain stem or part of the cerebellum

pain in head, neck, limbs, trunk made worse by straining cerebellar symptoms e.g. vertigo, gait disturbance

Signs: decerebrate posture, cardiorespiratory failure, death, pyramidal signs

84
Q

What causes raised intracranial pressure and what are the symptoms?

A

Space Occupying Lesions (tumour, haematoma, abscess) Idiopathic Intracranial Hypertension

Headache, nausea, visual disturbance, later altered consciousness levels

Papilloedema , increased blind spot on visual field testing

85
Q

What is Arnold-Chiari malformation and what are the symptoms?

A

Congenital malformation. Displacement of the cerebellar tonsils down through the foramen magnum. Sometimes blocks CSF flow causing hydrocephalus.

Symptoms can include headaches, fatigue, muscle weakness in the head and face, difficulty swallowing, dizziness, nausea, impaired coordination. Patients may also develop syringomyelia

86
Q

What is the peripheral nervous system?

A

Neurones of the PNS transmit signals better the CNS and receptors and effectors in all other parts of the body
Peripheral nerves can contain nerve fibres that are the axons of efferent neurones (motor), afferent neurones (sensory) or both - so fibres in a nerve can be classified as belonging to the efferent or afferent branch of the PNS
All of the spinal nerves contain both the efferent and afferent branches, whereas some cranial nerves contain only afferent fibres (optic nerve) or only efferent fibres (hypoglossal nerve)

87
Q

What are afferent neurones?

A

Convey information from sensory receptors at their peripheral ending to the CNS
The long part of their axon lies OUTSIDE of the CNS and is part of the PNS
Sometimes called fist-order neurones because they are the first cells entering the CNS in the synaptically linked chains of neurones that handle incoming information

88
Q

What are the efferent neurones?

A

Carry signals out from the CNS to muscles, glands and other tissues
Subdivided into somatic nervous system (voluntary) and the autonomic nervous system, both of these terms together make up the efferent division of the PNS

89
Q

Describe the somatic nervous system?

A
  • Innervate skeletal muscle
  • Consist of a single neurone between the CNS and skeletal muscle cells
  • Consist of a single neurone between the CNS and skeletal muscle cell
  • Can ONLY lead to muscle excitation
  • Conscious of its actions - voluntary
  • Made up of all the nerve fibres going from the CNS to skeletal muscle cells
  • The cell bodies of these neurones are located in groups in the brainstem or the ventral horn of the spinal cord
  • Have large-diameter myelinated axons which leave the CNS and pass without ANY SYNAPSE directly to the skeletal muscle cells
  • Since the activity of somatic neurones results in contraction of the innervated skeletal muscles cells, these neurones are called motor neurones
  • Only neurotransmitter involved is acetyl-choline (ACh)
90
Q

Describe the autonomic nervous system?

A
  • Innervate smooth & cardiac muscle, glands, neurones in the GI tract (enteric nervous system) and other tissues
  • Has a two-neurone chain (connected by synapse) between the CNS and the effector organ
  • The first neurone has its cell body IN the CNS
  • The synapse between the neurones is outside the CNS in a cell cluster called an autonomic ganglion
  • The neurones passing between the CNS and the ganglia are called preganglionic neurones, those passing between the ganglia and the effector cells are postganglionic neurones
  • Can either by excitatory or inhibitory
  • Function without conscious awareness (involuntary)
  • Acetyl-choline (ACh) is used before ganglion, then either acetylcholine (excitatory) or noradrenaline (inhibitory) is used after ganglion
91
Q

What are the features and effects of the sympathetic (fight or flight) nervous system?

A

Features:
• Leaves the CNS from the thoracic & lumbar regions (T1 - L2) of the spinal cord
• Most of the ganglia lie close to the spinal cord and form two chains of ganglia - one on each side of the cord - known as the SYMPATHETIC TRUNKS
• Uses acetyl-choline (ACh) at preganglionic synapse where there are nicotinic receptors
• At effector cell synapse the neurotransmitter noradrenaline is used where there are adrenergic receptors (of which there are 5 subtypes)
• Preganglionic fibres form the lateral grey horn
• Supplies visceral organs and structures of superficial body regions
• Contains more ganglia than the the parasympathetic division
• Its effects are amplified by the adrenal glands (FORMED BY THE SYMPATHETIC DIVISION) which in turn release adrenaline directly into the blood resulting in a high blood pressure & heart rate

Effects:
• Increases heart rate 
• Increases force of contractions in the heart 
• Vasoconstriction 
• BronchoDILATION 
• Reduces gastric motility 
• Sphincter contraction 
• DECREASED gastric secretions 
• Male ejaculation
92
Q

What are the features and effects of the parasympathetic (rest and digest) nervous system?

A

Features:
• Leaves the CNS from the brainstem & sacral portion of the spinal cord
• Cranial outflow: - Comes from brain - Preganglionic fibres run via; oculomotor nerve CN3 - to pupil, facial nerve CN7 - to salivary glands, glossopharyngeal nerve CN9 - for swallowing reflex & via the vagus nerve CN10 - to thorax & abdomen - remember by 1973 - Cell bodies are located in cranial nerve nuclei in the brainstem - Innervates the organs of the head, neck, thorax & abdomen • Sacral outflow: Supplies the remaining abdominal and pelvic organs
• Ganglia lie within/very close to the organs that the postganglionic neurones innervate
• Uses acetyl-choline (ACh) at the preganglionic neurone synapse where there are nicotinic receptors
• Uses acetyl-choline (ACh) at the effector cell synapse where there are muscarinic receptors

Effects:
• Decreases heart rate 
• Decrease force of contraction 
• Vasodilation 
• BronchoCONSTRICTION
 • Increases gastric motility 
• Sphincter relaxation 
• INCREASED gastric secretions 
• Male erection
93
Q

What is the enteric nervous system?

A

Nervous system of the GI tract

Can operate independently of the autonomic nervous system

94
Q

Describe Motor Control?

A

Muscles can only contract or relax (i.e. stop contracting)
The activation of muscle fibres is all or none
A skeletal muscle is attached to the bone by a tendon
A skeletal muscle comprises of several muscle fasiculi (group of muscle fibres)
A muscle fasiculus comprise several muscle fibres
A muscle fibre is constituted of several myofibrils
Myofibrils contain protein filaments; actin & myosin myofilaments
When the muscle fibre is depolarised actin and myosin slide against each other and produce muscle contraction

95
Q

What is a motor unit?

A

An ALPHA MOTOR NEURONE (type of lower motor neurone) and all the EXTRAFUSAL skeletal muscle fibres it innervates
The muscle fibers in a single motor unit are located in one muscle but they are distributed throughout the muscle and are not necessarily adjacent to each other (see diagram)
Different motor neurones innervate different numbers of muscle fibres, the less fibres that are innervated, the greater the variation of movement e.g. in the finger tips & tongue
Activation of an alpha motor neurone depolarises and cause contraction of all the fibres in that unit
The more a motor neurones fire, the more fibres contract resulting in more power
Alpha motor neurones controlling distal muscles are located laterally in the spinal cord
Alpha motor neurones controlling proximal muscles are located medially in the spinal cord

96
Q

Describe the neuromuscular junction?

A
  • The myelin sheath surrounding the axon of each motor neurone ends near the surface of a muscle fibre, and the axon divides into a number of short processes that lie embedded in the grooves on the muscle fibre surface
  • The axon terminals of a motor neurone contain vesicles similar to those found in the synaptic junctions between two neurones
  • The vesicles contain the neurotransmitter acetyl-choline (ACh) • The region of the muscle fibre plasma membrane that lies directly under the terminal portion of the axon is known as the MOTOR END PLATE
    • The junction of an axon terminal with the motor end plate is known as the neuromuscular junction
  • When an action potential in a motor neurone arrives at the axon terminal it depolarises the plasma membrane, opening voltage-gated Ca2+ channels and allowing Ca2+ ions to diffuse into the axon terminal from the extracellular fluid
  • This Ca2+ binds to proteins that enable the membranes of the acetyl-choline containing vesicles to fuse with the neuronal plasma membrane, thereby releasing ACh into the extracellular cleft separating the axon terminal and the motor end plate
  • ACh diffuses from the axon terminal to the motor end plate where it binds to cholinergic nicotinic receptors
  • The binding of ACh opens an ion channel in the receptor protein (both sodium & potassium can pass through these channels)
  • Due to the differences in the electrochemical gradients across the plasma membrane, MORE Na+ moves in than K+ out - producing a local depolarisation of the motor end plate - the end-plate potential (EPP) (this is equivalent to an EPSP (excitatory postsynaptic potential) at a neuron-neuron synapse
  • However, the magnitude of a single EPP is much larger than that of an EPSP because the neurotransmitter is released over a larger surface area, thereby binding to many more receptors and opening many more ion channels
  • Thus, one EPP is more than sufficient to depolarise the muscle plasma membrane adjacent to the end-plate membrane to its threshold potential thereby initiating an action potential • This action potential is then propagated over the surface of the muscle fibre and into the T-tubules (to spread throughout the muscle)
  • Most neuromuscular junctions are located near the middle of a muscle fibre with newly generated action potentials propagating from this region in both directions towards the ends of the fibre
  • EVERY ACTION POTENTIAL in a motor neurone normally produces an action potential in each muscle fibre in its motor unit - in contrast to synaptic junctions where multiple EPSPs must occur in order for threshold to be reached and an action potential elicited in the postsynaptic membrane
  • ALL NEUROMUSCULAR JUNCTIONS are EXCITATORY
  • The synaptic junction of the neuromuscular junction also contains the enzyme acetylcholinesterase which breaks down ACh - the choline is then transported back into the axon terminals where it can be used for the synthesis of new Ach
97
Q

Describe stretch receptors in the muscle spindle?

A

Contain stretch receptors that monitor muscle LENGTH & rate of change of muscle length
- these receptors consist peripheral endings of afferent nerve fibers wrapped around modified muscle fibers
- the entire apparatus is known as a MUSCLE SPINDLE
- Receptors in the spindle will detect stretch regardless of the current muscle length
- The modified muscle fibres within the spindle are known as INTRAFUSAL FIBRES - The two ends of the intrafusal fibres are innervated by GAMMA MOTOR NEURONES (type of lower motor neurone)
- Gamma motor neurones keep the intrafusal fibres set at a length that optimises muscle stretch detection
- The two ends of the muscle spindle are contractile whilst the central portion is non-contractile
- There are two types of stretch receptor in the spindle:
• Nuclear chain fibers - responds to how much muscle is stretched
• Nuclear bag fibers - responds to both the magnitude of stretch and the speed with which it occurs
The middle third of the spindle is associated with fast type 1a afferent sensory nerves
- The inferior and superior thirds of the spindle are associated with slower conducting type 2 afferent sensory nerves
- Muscle spindles are attached by connective tissue in parallel to the extrafusal fibers
- An external force stretching the muscle also pulls on the intrafusal fibers, stretching them and activating their receptor endings
- The more or faster the muscle is stretched - the greater the rate of receptor firing
- If action potentials along motor neurons cause the contraction of extrafusal fibers, the resultant shortening of the muscle removes tension on the spindle and thus slows the rate of firing in the stretch receptor
- thereby resulting in a reduction of sensory information

  • To counter this alpha-gamma coactivation is used to prevent this loss of information:
  • The contractile ends of the intrafusal fibers are too small & weak to contribute to force or shortening of the entire muscle
  • However, they can maintain tension and stretch in the central receptor region of the intrafusal fibre
  • Thus activating the gamma-motor neurones ALONE will increases the sensitivity of the muscle to stretch
  • Coactivating BOTH the alpha and gamma motor neurones will prevent the central region of the muscle spindle from going slack during a shortening muscle contraction
  • ensuring that information about muscle length will be continuously available to provide for adjustment during ongoing actions and to plan and program future movements
98
Q

Describe tension receptors in the muscle spindle?

A

Tension depends on muscle length, the load on the muscles & the degree of muscle fatigue - thus feedback is necessary to inform the motor control system of the tension actually achieved

  • Some of this feedback is provided by vision (you can see whether you are lifting or lowering an object) as well as by afferent input from the skin, muscle & joint receptors
  • Another receptor type called the GOLGI TENDON ORGANS specifically monitors how much TENSION the contracting motor units are exerting (or is being imposed on the muscle by external forces if the muscle is being stretched)
  • they measure the force developed by the muscles & any resultant change in length
  • Golgi tendon organs are endings of afferent fibres that wrap around collagen bundles in the tendons near their junction with the muscles
  • these collagen bundles are slightly bowed in their resting state
  • The afferent fibres leading from the golgi tendon organ to the spinal cord are 1b fibres that run to the anterior horn of the spinal cord
  • When the muscle is stretched or the attached extrafusal muscle fibres contract, tension is exerted on the tendon
  • this tension straightens the collagen bundles and distorts the golgi tendon receptor endings thereby activating them
  • NOTE: the tendon is stretched much more by an active contraction of the muscle than when the whole muscle is passively stretched
  • thus golgi tendon organs discharge in response to the tension generated by the contracting muscle and initiate action potentials that are transmitted to the CNS
  • Branches of the afferent neurone (1b fibres) from the golgi tendon organ cause the inhibition of ALPHA MOTOR NEURONES of the contracting muscle (thereby inhibiting muscle contraction) and its synergists (muscles whose contraction assists the intended motion) via interneurons in order to regulate muscle tension at a normal range and to protect it from overload - known as the INVERSE STRETCH (MYOTATIC) REFLEX

The output produced by the golgi tendon is proportional to muscle tension (force) - Golgi tendon organs possess slower afferent fibres than muscle spindles - They also stimulate the motor neurones of the antagonistic muscles - NOTE: things called Pacinian corpuscles (occur in the skin & deep tissue) detect vibrations

99
Q

Describe the stretch/myotatic reflex?

A

Crucially important for the control of skeletal muscle tone

  • When the afferent fibers from the muscle spindle enter the CNS they divide into branches that take different paths
  • From the diagram, path A, makes an excitatory synapse directly onto motor neurones which return to the muscle that was stretched
  • completing a reflex arc called the stretch reflex
  • A good example of this reflex is the knee jerk reflex
  • whereby the patellar tendon is tapped, as the tendon is pushed in by tapping, the thigh muscles it is attached to are stretched and all the stretch receptors within these muscles are activated
  • This stimulates a burst of action potentials in the afferent nerve fibers from the stretch receptors of the muscle spindle which in turn activate excitatory synapses on the motor neurones that control these same muscles
  • This results in the stimulation of the motor units of the thigh muscles causing them to contract resulting in the extension of the lower leg - the knee jerk
  • The presence of the knee jerk is normal and indicates that the afferent fibers, balance of synaptic input to the motor neurones, motor neurones themselves, neuromuscular junctions and the muscles themselves are functioning properly
  • Since the afferent fibres synapse directly with the motor neurones without any interneurones this portion of the stretch reflex is called MONOSYNAPTIC
  • stretch reflexes have the only known monosynaptic reflex arcs
  • All OTHER REFLEX ARCS are POLYSYNAPTIC
  • they have at least one interneurone, and usually many, between the afferent & efferent neurones
  • In path B, the branches of the afferent nerve fibres from stretch receptors end on inhibitory interneurons
  • when activated these inhibit the motor neurons controlling antagonistic muscles whose contraction would interfere with the reflex response. In the knee jerk
  • neurons to muscles the flex the knee are inhibited (this component of the stretch reflex is polysynaptic)
  • The activation of neurones to one muscle with the simultaneous inhibition of neurones to its antagonists muscle (as mentioned above) is known as reciprocal innervation
  • In path C, motor neurones of synergistic muscles (those whose contraction assist the intended motion) are activated, in the knee jerk, this would include the other muscles that extend the leg
  • NOTE: from the diagram, path D, shows that information about changes in muscle length ascend to higher centres, this information is especially important during slow, controlled movements such as the performance of unfamiliar actions. The information also contributes to proprioception - Since stretch reflexes operate to maintain muscles at a constant length in opposite to imposed stretch, they are important in the control of body posture
100
Q

Describe the withdrawal reflex?

A

In addition to the afferent information transmitted from the spindle stretch receptors and golgi tendon organs, other input is transmitted to the local motor control system

  • Painful stimulation, e.g stepping on a pin, on the skin, activates the flexor muscles and inhibits the extensor muscles of the ipsilateral (on the same side of the body) leg
  • this results in the affected limb moving away from the harmful stimulus
  • this is the withdrawal reflex
  • At the same time, the stimulus causes the opposite response on the contralateral (on the opposite side of the body to the stimulus) leg; motor neurones to the extensors are activated while the flexor muscle motor neurones are inhibited
  • this is known as the crossed-extensor reflex and enables the contralateral leg to the support the body’s weight as the injured foot is lifter by flexion
101
Q

What is muscle tone?

A
  • Muscle tone is the degree of contraction of a muscle or the proportion of motor units that are active at any one time
  • A muscle with high tone feels firm, or rigid and resists passive stretch
  • A muscle with low tone feels soft or flaccid and offers little resistance to passive stretch
  • When a person is very relaxed the alpha motor neurone activity does not make a significant contribution to the resistance to stretch
    • As a person becomes increasingly alert, more activation of the alpha motor neurons occur and muscle tone increases
102
Q

What is abnormal muscle tone?

A
  • Abnormally high muscle tone is called HYPERTONIA and accompanies a number of diseases and is seen very clearly when a joint is moved passively as high speeds
  • the increased resistance is due to an increased level of alpha motor neurone activity which keep the muscle contracted despite the attempt to relax it
  • Hypertonia usually occurs with disorders of the descending pathways which normally inhibit the motor neurones
  • The descending pathways and neurones of the motor cortex are referred to as the UPPER MOTOR NEURONES - abnormalities due to their dysfunction are referred to as upper motor neurone disorders
    Thus hypertonia indicates an upper motor neurone disorder
  • Alpha motor neurones are referred to as LOWER MOTOR NEURONES
    Spasticity is a form of hypertonia in which the muscles do not develop increased tone UNTIL they are stretched a bit, and after a brief increase in tone, the contraction subsides for a short time -
    The period of ‘give’ occurring after a time of resistance is called the CLASPKNIFE PHENOMENON - essentially when someone bends the limb of a patient, initially there is some resistance but after a certain point resistance falls dramatically
    The clasp-knife reflex is caused by the 1b afferent fibres from the golgi tendon organs INHIBITING alpha motor neurones once the golgi tendon organs detect tension - it is an example of the inverse stretch (myotatic) reflex
    The clasp-knife reflex is characteristic of an UPPER MOTOR NEURONES LESION
    RIGIDITY is a form of hypertonia in which the increased muscle contraction is continual and the resistance to passive stretch is constant
    HYPOTONIA is a condition of abnormally low muscle tone accompanied by weakness, atrophy (decrease in muscle bulk) and decreased/absent reflex responses
    it may develop after cerebellar disease but also commonly accompanies disorders of the alpha motor neurones, LOWER MOTOR NEURONE DISORDERS/LESIONS, neuromuscular junctions or the muscles themselves
103
Q

What is myasthenia Gravis and what are the symptoms?

A

Autoimmune
• Muscle weakness caused by circulating antibodies that block the acetylcholine receptors at the postsynaptic side of the neuromuscular junction
• This blocks the excitatory effect of ACh on the nicotinic receptors resulting in muscle weakness
• More common in women

Symptoms: 
Face getting progressively droopy in the morning 
- Tiring or difficult to chew food 
- Double vision
 - Eyelid drooping
104
Q

What is Duchenne Muscular Dystrophy and what are the symptoms?

A
  • X-linked recessive
  • Only affects boys
  • Results in muscle degeneration and eventually premature death
  • Affects MUSCLES

Symptoms:

  • Awkward manner of running with frequent falls & more easily fatigued
  • Difficulty with motor skills e.g. running & jumping
  • Typically in very young boys since most die before they get too old
  • Boys will find it difficult to get into standing position
105
Q

What basic components do all neurones have?

A

Dendrites
Cell body/soma
Axon
Presynaptic terminal

106
Q

Describe histology of neurones?

A

Under H & E the haemotoxylin stains the nucleic acids are stained blue, and the eosin stains the proteins red

Luxor fast blue (LFB) stains myelin

Cresol violet (CV) stains Nissl (RER)

107
Q

What is neural plasticity?

A

the basis of learning & memory

108
Q

What is an early marker of alzheimers in neurones?

A

Loss of dendritic spines

109
Q

What are the three functional classes of neurones?

A

Afferent, Efferent, Interneurons

110
Q

What is an afferent neurone?

A

Convey information from tissues and organs TOWARDS the CNS - At their peripheral ends (farthest from the CNS), afferent neurons have sensory receptors which respond to various physical or chemical changes in their environment by generating electrical signals in the neurone - Afferent neurons have a distinct shape, shortly after leaving the cell body, the axon divides; one branch (the peripheral process) begins where the dendritic branches converge from the receptor endings, the other branch (the central process), enters the CNS to from junctions with other neurons - The cell body & long axon (peripheral process) are OUTSIDE the CNS, and only a part of the central process enters the brain or spinal cord

111
Q

What is an efferent neurone?

A

Convey information AWAY from the CNS to effector cells such as muscle, gland or other cell types
Cell bodies & dendrites are WITHIN the CNS, and the axons extend OUT to the periphery

112
Q

What is an interneuron?

A

Connect neurons WITHIN the CNS - form the majority of neurons - Lie entirely WITHIN the CNS

113
Q

Describe myelination?

A

The axons of many neurons are covered by sheaths of myelin which usually consist of 20 to 200 layers of highly modified plasma membrane wrapped around the axon by a nearby supporting cell
Highly compacted - 70% lipid & 30% protein
In the brain & spinal cord these myelin forming cells are the oligodendrocytes - each oligodendrocyte may branch to form myelin on as many as 40 axons
In the PNS, cells called Schwann cells form individual myelin sheaths surrounding 1 to 1.5 mm-long segments at regular intervals along the axons
The spaces between adjacent sections of myelin where the axons plasma membrane is exposed to extracellular fluid are called the node of Ranvier
Myelin increases the speed of conduction along the axons
Myelinated axons are thicker and are mostly found in somatic nerves i.e. in fast sensory/motor systems e.g. muscle & spinal systems
Unmyelinated axons tend to be thinner and found in post-ganglionic autonomic fibres, fine sensory fibres, olfactory neurones & interneurons - essentially where speed is not of the essence e.g. hypothalamus (hormonal)

114
Q

What are glial cells?

A

Glial cells surround the soma (cell body), axon and dendrites of neurones and provide them with physical & metabolic support

115
Q

Name the glial cells?

A

Oligodendrocytes, Schwann Cells, Astrocytes, Muller Glia, Radial Glia, Bergmann glia, Ependymal Cells, Microglia

116
Q

What are oligodendrocytes?

A

Glial cells of the CNS
• Myelinating cells
• Myelin insulates axon segments enabling rapid nerve conduction
• Also provide metabolic support for axons, they are able to transport metabolic products directly into axons
• They myelinate MULTIPLE axons

117
Q

What are Schwann Cells?

A
  • Glial cells of the PNS
  • Myelinating cells
  • They myelinate SINGLE axons
118
Q

What are astrocytes?

A

• Help regulate the composition of the extracellular fluid in the CNS by removing K+ ions & neurotransmitters (e.g. glutamate) around synapses
• Astrocytes take up glutamate then convert it to glutamine and release it, then neurones can take it up and convert it back to glutamate for reuse
• They also stimulate the formation of tight junctions between the cells that make up the walls of capillaries found in the CNS - this forms the BLOOD-BRAIN BARRIER, which is a much more selective filter for exchanged substances that is present between the blood and most other tissues
• They help form the blood-brain barrier by foot-processes closely applied around capillaries
They also sustain the neurones metabolically e.g. by providing glucose and removing ammonia
• Star-like cells
• MOST NUMEROUS glial cells in the CNS
• Protoplasmic - found in grey matter
• Fibrous - found in white matter

119
Q

What are microglia?

A

Specialised macrophage-like cells that perform immune functions in the CNS
• Derived from progenitors that migrate into the CNS from the periphery
• Proliferate at sites of injury - phagocytic
• Migrate to sites of damage
• In the cortical grey matter they are more ramified (branched)
• They phagocytose debris/microbes
• They contribute to synaptic plasticity i.e. microglia can “eat” unwanted dendritic spines
• They can be bad, by being too sensitive causing excessive inflammation & destruction of dendritic spines

120
Q

What are ependymal cells?

A
  • Line fluid-filled cavities within the brain (i.e. ventricles) and spinal cord and regulate the production & flow of cerebrospinal fluid (CSF)
  • Have cilia, microvilli & desmosomes
  • Provides barrier between CSF & brain
121
Q

What are radial glia?

A

crucial in guiding developing neurones - these are only developmental and are not found in the adult brain

122
Q

What are muller glia?

A

specialised radial glia of the retina

123
Q

What are Bergmann glia?

A

found in the cerebellum, support purkinje cell dendrites & synapses

124
Q

Which conditions affect neurones and glia?

A

Motor Neurone Disease
Depression
Alzheimer’s Disease
Multiple Sclerosis

125
Q

Which condition affects only neurones?

A

Epilepsy

126
Q

How is a resting potential achieved in a neurone?

A

All cells under resting conditions have a potential difference across their plasma membranes, with the inside of the cell negatively charged with respect to the outside. This is known as the resting membrane potential
The magnitude of the resting potential in neurons varies from about -40 to -90mV. The typical resting potential is usually – 70 mV
Na+ and K+ ATPase pumps in the neurone membrane develop concentration gradients by pumping 3 Na + ions out of the neurone for every 2 K+ ions that are pumped in, against both ions respective concentration gradients, via active transport
- Consequently, Na + ions are concentrated outside the axon membrane and K+ ions are concentrated inside
- Very few Na+ voltage gated channels are open at this point meaning few Na+ ions can diffuse back into the axon, K+ voltage gated channels are also closed but K+ channels (leak K+ channels) are open
– increasing the membranes permeability to K+ ions - This results in K+ ions diffusing out of the axon, down their concentration gradient, making the inside of the axon more negative than outside
- Eventually the electrochemical gradient, which will pull the K+ ions into the axon, will reach equilibrium with the concentration gradient, which will push K+ ions out of the axon. This equilibrium is reached at -70 mV and the resting potential is reached

127
Q

Describe how an action potential is formed?

A
  • When a neurotransmitter binds to specific ligand-gated ion channels on the post-synaptic membrane, Na+ ions are allowed to enter the neurone
  • The inflow of Na+ ions results in the inside of the neurone to become slightly more positive – initial depolarization
  • This initial depolarization stimulates the opening of some voltage-gated Na+ channels, resulting in further entry of Na+ ions into the neurone and thus further depolarization
  • When the membrane reaches the critical threshold potential (about -55mV), depolarization becomes a positive feedback loop

– Na+ entry causes depolarization, which opens more voltage gated Na+ channels, which results in more depolarization, and so on

  • When the membrane potential reaches +30 mV (sometimes referred to as reverse polarization), the voltage-gated Na+ channels are inactivated and Na+
    influx stops - The “sluggish” voltage-gated K+ channels, open in a delayed response to the initial depolarization, this results in K+ diffusing out of the neurone, down their concentration gradient, causing the neurone to rapidly repolarize back to its resting potential - The return of the neurone to a negative potential causes the voltage-gated K+ channels to close, however due to their sluggish nature they do this slowly

This means that the membranes permeability to K+ remains above resting levels, resulting in the continued outflow of K+, making the inside of the neurone much more negative than -70mV – hyperpolarization - Once the voltage-gated K+ channels finally close, the resting potential is restored

128
Q

What is the absolute refractory period?

A

During the action potential, a second stimulus, no matter how strong, will not produce a second action potential, since that region of the neurone membrane is in its absolute refractory period
this occurs during the period when the voltage-gated Na+ channels are either already open or have proceeded to their inactivated state after during the first action potential

129
Q

What is the relative refractory period?

A

Following the absolute refractory period, there is an interval whereby a second action potential can be produced but only if the stimulus strength is considerably greater than usual. this is the relative refractory period (lasts until the membrane returns to the resting potential) - The refractory periods limit the number of action potentials an excitable membrane can produce in a given period of time. They allow action potentials to be separated so that individual electrical signals are able to pass down the axon

130
Q

Describe conduction in the neurones?

A
  • The generation of an action potential at particular segment on the neurone membrane causes a current to flow, due to the difference in potential between the depolarized membrane and adjacent segments at resting potential
  • This current, depolarizes adjacent membrane where it causes voltage-gated Na+ channels located there to open – resulting in an action potential - The current entering during an action potential is sufficient to easily depolarize the adjacent membrane to the threshold potential
  • The new action potential generates local currents of its own, that depolarize the region adjacent to it, producing another action potential at the next site, and so on, to cause action potential propagation along the length of the membrane
  • Propagation travels in the direction of the region of membrane that has recently been active, the impulse isn’t able to travel backwards since the adjacent membrane behind the action potential will be in its absolute refractory period and thus will be unable to depolarize
131
Q

Describe propogation speeds in axons?

A
  • Propagation along a membrane depends on fibre diameter & myelination
  • The larger the fibre diameter, the faster the action potential propagates, since a larger fibre offers less internal resistance to local current meaning adjacent regions of the membrane are able to reach threshold faster
    Myelination increases propagation speeds, this is because there is less “leakage” of charge across the myelin meaning a local current can spread farther along an axon. Also the concentration of Na+ channels in the myelinated region of the axon is low. Therefore, action potentials only occur at the nodes of Ranvier, where the myelin coating is interrupted and the concentration of voltagegated Na+ channels is high
    Action potentials appear to jump from one node to the next as they propagate along a myelinated fibre, this is known as SALTATORY CONDUCTION
    Propagation via saltatory conduction is faster that propagation in non-myelinated fibres of the same axon diameter - Conduction velocities vary from around 0.5m/s in small-diameter unmyelinated fibers to about 100m/s in large-diameter myelinated fibers
132
Q

Explain Multiple Sclerosis and it’s symptoms?

A

Axonal Transmission Failure

  • Axonal transmission is the transmission of information from point A to B
  • Multiple sclerosis is the most common disease of the nervous system amongst young adults
  • Thought to be an autoimmune disease
  • It is the degeneration of myelin and development of scar tissue which in turn disrupts and eventually blocks neurotransmission along myelinated axons
Symptoms: 
• Uncontrolled eye movements - seeing double 
• Slurred speech
 • Partial/complete paralysis
 • Tremor 
• Loss of co-ordination 
• Weakness 
• Sensory numbness, prickling, pain
133
Q

What is a synapse?

A

An anatomically specialised junction between two neurones at which the electrical activity in a presynaptic neurone influences the electrical activity of a postsynaptic neurone

134
Q

What is an excitatory synapse?

A

The membrane potential of a postsynaptic neurone is brought closer to threshold (depolarised) at an EXCITATORY SYNAPSE
If the membrane of a postsynaptic neurone reaches threshold, it will generate action potentials that are propagated along its axon to the terminal branches which in turn influence the excitability of other cells

135
Q

What is an inhibitory synapse?

A

The membrane potential of a postsynaptic neurone is either driven further from threshold (hyperpolarised) or stabilised at its resting potential at an INHIBITORY SYNAPSE
If the membrane of a postsynaptic neurone reaches threshold, it will generate action potentials that are propagated along its axon to the terminal branches which in turn influence the excitability of other cells

136
Q

Desribe electrical synapses?

A
  • The plasma membranes of the presynaptic and postsynaptic cells are joined by gap junctions
  • These allow the local currents resulting from arriving action potentials to flow directly across the junction through the connecting channels from one neurone to another
  • This depolarises the membrane of the second neurone to threshold, contingent the propagation of the action potential
  • Communication between cells via electrical synapses is extremely rapid
  • They allow for synchronised transmission
  • Found in brainstem neurons e.g. breathing & hypothalamus e.g. hormone secretion
137
Q

Describe chemical synapses?

A
  • The plasma membranes of presynaptic and postsynaptic neurones are joined by the synaptic cleft
  • The axon of the presynaptic neurone ends in a slight swelling, the axon terminal, which holds the synaptic vesicles that contain neurotransmitter molecules
  • The synaptic cleft separates the presynaptic and postsynaptic neurons and prevents direct propagation of the current from the presynaptic neurone to the postsynaptic cell
  • Instead, signals are transmitted across the synaptic cleft by means of a chemical messenger known as a neurotransmitter which is released by the presynaptic axon terminal
  • Occasionally, more than one NEUROTRANSMITTER may be simultaneously released from an axon, in which case the additional neurotransmitter is called a cotransmitter
  • NOTE: neurotransmitters have different receptors on the postsynaptic cell
  • Synapses are covered by astrocytes (glial cell) which is essential for the reuptake of excess neurotransmitter
138
Q

Describe neuroatransmitter release?

A
  • Calcium ion CHANNELS open when an action potential reaches the presynaptic terminal
  • Ca2+ ions cause vesicles (containing neurotransmitter) to move to release sites and fuse with the presynaptic cell membrane and discharge their contents
  • Neurotransmitter diffuses across the synaptic cleft and attaches to receptor sites on the post-synaptic membrane
  • NOTE: the higher the concentration of neurotransmitter released, the more likely there will be binding to receptor and thus action potential propagation
139
Q

What are the 5 processes of synaptic transmission?

A
  1. Manufacture Intracellular biochemical processes
  2. Storage Vesicles
  3. Release Action Potential
  4. Interaction with post-synaptic receptors Diffusion across synapse
  5. Inactivation Break down or reuptake
140
Q

Describe neurotransmitter effect on the postsynaptic membrane?

A
  • Once neurotransmitters have been released, only a fraction of them bind to receptors on the postsynaptic neurone - these receptors can take the form of transmitter-gated ion channels
  • These channels are sensitive to SPECIFIC neurotransmitters
  • When they bind to the channels it results in depolarisation or hyper polarisation depending on the channel type; - Depolarisation will occur in excitatory channels (excitatory postsynaptic potential (EPSP)) - Hyperpolarisation will occur in inhibitory channels (inhibitory postsynaptic potential (IPSP)) - many K+ leave OR many Cl- enter
  • Once a neurotransmitter has bound then it will result in the propagation of action potentials (if excitatory channel)
141
Q

Describe temporal summation?

A

Input signals arrive from the same presynaptic cell at different TIMES. The potentials summate since there are a greater number of open ion channels and thus a greater flow of positive ions into the cell

142
Q

Describe spatial summation?

A

Where two inputs occur at different locations in the postsynaptic neurone

143
Q

What happens to neurotransmitter that are unbound?

A

Unbound neurotransmitters are removed from the synaptic cleft when:

1 They are actively transported back into the presynaptic axon terminal (through a process called reuptake) or in some cases by nearby glial cells
2 They diffuse away from the receptor site
3 Are enzymatically transformed into inactive substances, some are transported back into the presynaptic neurone for reuse

144
Q

What are fast neurotransmitters?

A

short lasting effects, tend to be involved in rapid communication

Acetylcholine (ACh) - Glutamate (GLU) (excitatory) - GABA (inhibitory)

145
Q

What are neuromodulators?

A

cause change in synaptic membrane that last for longer time e.g. minutes, hours or even days, include alterations in enzyme activity or influences DNA transcription in protein synthesis. Associated with slower events such as learning, development, motivational states etc

Dopamine (DA) - Noradrenalin (NA) or Norepenephrin - Serotonin

146
Q

How do local anaesthetics work?

A
  • Most common are procaine & lignocaine
  • These local anaesthetics work by interrupting axonal neurotransmission
  • They do this by blocking sodium channels thereby preventing the neurones from depolarising meaning threshold isn’t met and thus no action potential is developed to be propagated
  • This results in pain relief since pain isn’t transmitted
  • Local anaesthetics can diffuse through mucus membranes EASILY thus sometimes can act on muscles too
147
Q

Describe acetylcholine?

A

Major neurotransmitter of the PNS at the neuromuscular junction

  • Also used in the brain & spinal cord
  • Neurones that release ACh are known as cholinergic neurons
  • Acetylcholine is synthesised from choline (common nutrient found in food) & acetyl coenzyme A in the cytoplasm of synaptic terminals and stored in synaptic vesicles
  • After it is released and activated receptors on the postsynaptic membrane, the concentration of ACh at the postsynaptic membrane decreases (thereby stopping receptor activation) due to the action of the enzyme acetylcholinesterase
  • Acetylcholinesterase is located on the postsynaptic & presynaptic membranes and rapidly destroys ACh, releasing choline & acetate - The choline is transported back into the presynaptic axon terminal where it is reused in the resynthesis of Ach
148
Q

What are the receptors for acetylcholine?

A
  • There are two general types of ACh receptors:
  • Nicotinic receptors: Since these respond not only to ACh but also nicotine. This receptor contains an ion-channel. Found in the NEUROMUSCULAR JUNCTION. Nicotinic receptors in the brain are important in cognitive functions & behaviour e.g. one cholinergic system that employs nicotinic receptors plays a major role in attention, learning and memory by reinforcing the ability to detect and respond to meaningful stimuli. The presence of nicotinic receptors on presynaptic terminals in reward pathways of the brain explains why tobacco products are so addictive
  • Muscarinic receptors: Since these respond not only to ACh but also to the mushroom poison muscarine. These receptors couple with G proteins, which in turn then alter the activity of a number of different enzymes & ion channels. These receptors are present in the brain and at junctions where a major division of the PNS innervates peripheral glands & organs e.g salivary glands & the heart & lungs (bronchoconstriction (M3)) -
149
Q

What is the clinical relevance of acetylcholine?

A

Cigarettes contain nicotine which are agonists - able to interact & open receptor -

Sarin, inhibits the action of acetylcholinesterase thereby causing a buildup of ACh in the synaptic cleft resulting in overstimulation of postsynaptic ACh receptors, initially causing uncontrolled muscle contractions but eventually leading to receptor desensitisation and paralysis

150
Q

Describe Noradrenaline and it’s clinical significance?

A

Transmitter in the peripheral heart and central nervous systems

-

Affected by:
• Antidepressant drugs: Imipramine (blocks the reuptake of noradrenaline. Therapeutic effect is only seen after 3-5 weeks, since the blockage of reuptake DOES NOT cause therapeutic effect, instead its the brains response to it that does)
• Antidepressant drugs: Monamine oxidase (MAO) inhibitor - increases the amount of noradrenaline by inhibiting the enzyme monoamine oxidase (MAO) which is the enzyme used to break down noradrenaline
• Stimulants: Amphetamine - Increases release & blocks reuptake

151
Q

Describe dopamine and its clinical relevance?

A

Important transmitter in the BASAL GANGLIA

-

Affected by:
• Antipsychotic drugs: Such as chlorpromazine which is an antagonist - blocks receptor so other neurotransmitter cannot activate receptor
• Stimulants: Amphetamine/cocaine - Increases releases & blocks reuptake
• Anti-parkinsons drug: L-DOPA increases dopamine manufacture

  • In parkinsons there is a degradation/death of dopaminergic neurones
  • L-DOPA is a precursor for dopamine
  • L-DOPA is given to patients, it is able to cross the blood brain barrier
  • It is taken up by serotonin neurones and converted & released as dopamine due to the fact that serotonin neurones contain the same enzyme needed to convert L-DOPA to dopamine as the dopaminergic neurones have
152
Q

Describe serotonin and its clinical relevance?

A

Has an excitatory effect on pathways that mediate sensations

-

Affected by:
• Antidepressant drug: Prozac - Selective Serotonin Reuptake Inhibitor (SSRI), resulting in an increase in the concentration of synaptic serotonin
• Ecstasy: Neurotoxic to serotonin neurones, destroy the terminal of axons

  • In parkinsons there is a degradation/death of dopaminergic neurones
  • L-DOPA is a precursor for dopamine
  • L-DOPA is given to patients, it is able to cross the blood brain barrier
  • It is taken up by serotonin neurones and converted & released as dopamine due to the fact that serotonin neurones contain the same enzyme needed to convert L-DOPA to dopamine as the dopaminergic neurones have
153
Q

Describe glutamate?

A

Main Excitatory Neurotransmitter

154
Q

Describe GABA?

A

Main INHIBITORY neurotransmitter

155
Q

Define Pain?

A

An unpleasant sensory & emotional experience associated with actual potential tissue damage, or described in terms of such damage

156
Q

Define Acute Pain?

A

Short term pain of less than 12 weeks

157
Q

Define Chronic Pain?

A

Continuous long term pain of more than 12 weeks

158
Q

Define nociceptive pain?

A

Pain that arises from actual or threatened damage to nonneuronal tissue and is due to the activation of nociceptors

159
Q

Define neuropathic pain?

A

Pain initiated or caused by a primary lesion/dysfunction of the nervous system e.g. due to spinal nerve root compression

160
Q

What is a 1st order/primary afferent neuron?

A

Enters the spinal cord through a spinal nerve, or the brainstem through the trigeminal nerve, on the same side of the body (ipsilaterally) as the peripheral receptor is located
• Remains ipsilateral and synapses with a second order neurone within the CNS

161
Q

What is a 2nd order neuron?

A
  • Cell body is located in the spinal cord/brainstem (exact location depends where sensory receptor is, i.e. brainstem if the face etc.)
  • Its axons CROSS over (decussates) to the other side of the CNS and ascends to the thalamus where it terminates
162
Q

What is a 3rd order neuron?

A

• Cell body is located within the thalamus and its axon projects to the somatosensory cortex - located in the post central gyrus of the parietal lobe of the cerebral hemisphere

163
Q

Describe nociceptors and their role?

A

Sensory neurons that are found in any area of the body that can sense pain either externally or internally:
• Externally: skin, cornea, mucosa
• Internally: viscera, joints, muscles & connective tissue etc
Cell bodies either reside in dorsal root ganglion (body) or trigeminal ganglion (face/head/neck)
When there is tissue damage, bradykinin & prostaglandin E2
these substances both reduce the nociceptive action potential threshold thereby increasing their sensitivity to stimuli
known as hyperalgesia
Most are poly-modal
thermal/chemical/mechanical pain

164
Q

Describe alpha delta fibres?

A

Thinly myelinated
- Carries; touch, pressure, temperature & FAST pain information
- Small diameter (1-5 micrometers)
- Conduction speed is medium (5-40m/s)
The A delta (primary afferent neurone) terminals release GLUTAMATE (FAST ACTING) as their neurotransmitter

165
Q

Describe C fibres?

A

Unmyelinated
- Carries; SLOW pain, temperature, touch, pressure, itch, postganglionic autonomic fibres information
- Smallest diameter (0.2-1.5 micrometers)
- Conduction speed is the slowest (0.5-2m/s)
C fibres (primary afferent neurone) terminals release GLUTAMATE & SUBSTANCE P (SLOW ACTING - involved in the mediation of dull aching pain) as their neurotransmitters

166
Q

Where do nociceptors synapse?

A

Nociceptors (A delta & C fibres - first-order neurones) synapse with secondary afferent neurones (second order) in the GREY MATTER of the DORSAL HORN of the spinal cord (which is divided up into Rexed laminae)
First order/primary afferent neurones transmit information into the dorsal root ganglion where they synapse with second order neurones at the substantia gelatinosa

167
Q

Describe the trigeminothalamic tract?

A

First order neurones enter at the pons and then descend to the medulla forming the spinal trigeminal tract - Receives contributions from the trigeminal, facial, vagus & glossopharyngeal nerves - Both of these tracts terminate at the thalamus (ventral posterior lateral nucleus)

168
Q

What is the role of the thalamus in nociception?

A
Midline, paired symmetrical structure in the brain 
All sensation (except olfactory) relay/pass through it
 Contains multiple nuclei: - Relay function - Association function
169
Q

What is the role of the insula in nociception?

A

Lies within the brain via the sylvian fissure
Where the degree of pain is judged
Contributes to the subjective aspect of pain perception

170
Q

What is the role of the cingulate gyrus in nociception?

A

Located on the medial aspect of the cerebral hemispheres
Linked with the limbic system which is associated with emotion formation and processing learning & memory
Involved in the emotional response to pain

171
Q

What is the role of the periaqueductal grey in nociception?

A
  • Located in the midbrain
  • The grey matter located around the cerebral aqueduct
  • Receives input from the somatosensory cortex
  • Part of the descending pain pathway
  • Contains a high concentration of opioid receptors & endogenous opioids
  • Under situations of extreme stress this pathway can be activated
  • Resulting in the modulation of afferent noxious transmission
  • It projects to the dorsal horn
  • Once activated, opioid receptors are activated resulting in a reduction in PRE-SYNAPTIC neuronal sensitivity (thereby reducing Substance P release) which in turn results in reduced pain sensation
  • Meaning less impulses travel up the first, second & third order neurones to the somatosensory cortex - meaning less pain is felt
  • Opioids such as morphine,methadone, codeine & oxycodone mimic this effect by binding to the opioid receptors in the periaqueductal grey thereby conferring profound analgesia
172
Q

What is analgesia?

A

the selective suppression of pain with-out effects on consciousness or other sensations

173
Q

What is anaesthesia?

A

the uniform suppression of pain - NO PAIN IS FELT AT ALL, and sometimes consciousness is lost (general anaesthesia)

174
Q

What is the Melzack-Wall Pain Gate theory?

A

states that non-painful input closes the “gate” to painful input, thereby preventing pain sensation from travelling to the somatosensory cortex to be perceived and thus felt

175
Q

What is substance P?

A
  • Peptide neurotransmitter involved in pain transmission
  • Also a vasodilator
  • Remains bound to receptors for longer time thereby transmitting long-lasting pain
176
Q

What are the different types of long term memory and where are they located?

A

Explicit Memory: conscious memory e.g. remembering an appointment time

Episodic (autobiographical): HIPPOCAMPUS & midbrain

Semantic: knowledge about stuff, Frontal temporal lobe

Implicit: unconscious memory e.g. sensory motor skills such as driving a car):
• Skills/habits - cerebellum & basal ganglia
• Conditioned reflexes - cerebellum & others

Emotional:
Amygdala(Receives highly processed information - Produces instinctive emotional output - Responsible for emotional memory - Responsible for FEAR)
Anterior Cingulate Gyrus is also responsible

177
Q

What is the limbic system?

A

Acts as an interface between the internal environment of the individual and the external environment
Essential for adaptive behaviour, emotional responsiveness and the ability to learn new responses based on previous experiences (memory)
Its two main functions are; learning & the regulation and translation of our emotional state into appropriate behaviour
• Located on the edge (or limbus) of the hemisphere
• Operates by influencing the endocrine system and the autonomic nervous system, and is highly interconnected with the brains pleasure centres (the nucleus accumbens - has a role in sexual arousal and the high experienced with recreational drugs)
• The various components of the limbic system are connected via the Papez Circuit - essentially the circuits that connect the various components of the limbic system -

178
Q

What does the limbic system consist of?

A

Cingulate gyrus
Hippocampus (involved in long term memory formation)
Parahippocampal gyrus
Anterior perforated substance
Septal nuclei
Uncus
Amygdala (important in motivational significant stimuli - related to fear or reward)

179
Q

Describe the Direct Pathway/Cortical Loop (i.e. Increasing/making movement)?

A

In each hemisphere the opposite half of the body is represented in a highly precise fashion - DIRECT PATHWAY/ CORTICAL LOOP (i.e. increasing/making movement) e.g. in eye movement
The PRIMARY MOTOR CORTEX sends EXCITATORY messages to the STRIATUM using the neurotransmitter GLUTAMATE
This excites the striatum resulting in it sending more INHIBITORY messages to the INTERNAL GLOBUS PALLIDUS & the PARS RETICULATA of the SUBSTANTIA NIGRA using the neurotransmitter GABA
This INHIBITS the internal globus pallidus resulting in it sending less INHIBITORY messages to the THALAMUS
This also INHIBITS the pars reticulata meaning it sends less INHIBITORY MESSAGES to the PARS COMPACTA of the SUBSTANTIA NIGRA: - This means the pars compacta is able to send more EXCITATORY messages to further excite the INHIBITORY pathway to the pars reticulata thereby increasing the transmission of the direct pathway
using the neurotransmitter DOPAMINE which binds to D1 RECEPTORS - This also means the pars compacta sends more INHIBITORY messages to INHIBIT the inhibitory pathway to the external globus pallidus from the putamen
using the neurotransmitter DOPAMINE which binds to D2 RECEPTORS, meaning the external globus pallidus is able to send INHIBITORY messages (using GABA) to the SUBTHALAMIC NUCLEUS which in turn INHIBITS the excitation of the pars reticulata which would result in NO DOPAMINE RELEASE
The inhibition of the pars reticulate also means that less INHIBITORY messages (using GABA) are sent to the SUPERIOR COLLICULUS meaning it is able to send more EXCITATORY messages to the primary motor cortex using the neurotransmitter glutamate
This means the THALAMUS is able to send more EXCITATORY messages to the PRIMARY MOTOR CORTEX using the neurotransmitter glutamate
The combined effect of the excitatory messages from both the thalamus & superior colliculus to the primary motor cortex results in MORE EYE MOVEMENT

180
Q

Describe the Indirect Pathway/subcortical loop i.e. Decreasing/stopping movement?

A

The PRIMARY MOTOR CORTEX sends an EXCITATORY message to the putamen using the neurotransmitter glutamate
• This causes the striatum to send more INHIBITORY messages to the external globus pallidus via GABA neurotransmitter
This inhibits the external globus pallidus meaning it is unable to inhibit the subthalamic nucleus
This means the subthalamic nucleus is able to send more EXCITATORY messages to the internal globus pallidus & pars reticulata of the substantia nigra via glutamate neurotransmitter: - This excites the internal globus pallidus resulting in the release of INHIBITORY messages to the thalamus via GABA neurotransmitter
This excites the pars reticulata resulting in the release of INHIBITORY messages to the superior colliculus via GABA neurotransmitter
This inhibits the thalamus & superior colliculus meaning they are UNABLE to EXCITE the PRIMARY MOTOR CORTEX resulting in NO EYE MOVEMENT
NOTE; Dopamine can be metabolised to melanin (pigment) + adrenaline + noradrenaline. Also melanocytes produce melanin in the skin

181
Q

What are basic principles of Parkinson’s and Huntington’s?

A

Parkinson’s Disease - lack dopamine

- Huntington’s Disease - lack GABA

182
Q

What is the mechanism of Parkinson’s Disease?

A

NOT ENOUGH DOPAMINE in the substantia nigra - loss of dopaminergic neurones
• Substantia nigra appears “faded” since there is less dopamine production meaning there will be less byproduct (black pigment - neuramelanin)
• Less dopamine means that the external globus pallidus will not be able to inhibit the subthalamic nucleus meaning it will in turn excite the internal globus pallidus resulting in the inhibition of the thalamus and thus a decrease in movement

183
Q

What are the symptoms of Parkinson’s Disease?

A

• Increased muscle tone - spasticity • Reduced movements •• Bradykinesia (slow movements): - Problems doing up buttons - Writing appears smaller - Walking deteriorates - small steps & dragging of a foot • Tremor at rest & may be on one side only • Muscle rigidity - pain

184
Q

What is the treatment for Parkinson’s?

A

Aims to correct dopamine deficiency by using L-dopa which can be used by serotonin neurones and converted into dopamine - However the dopaminergic neurones will continue to die - The drug will work for shorter and shorter - An alternative is deep brain stimulation of the SUBTHALAMIC NUCLEUS which essentially inhibits the subthalamic nucleus meaning that the thalamus does not get inhibited and can thus excite the cortex resulting in improved movement

185
Q

What is the mechanism of Huntington’s Disease?

A

TOO LITTLE GABA resulting in too much dopamine
Autosomal dominant - with full penetrance
Atrophy (wasting) of the ventricles in the brain result in the enlargement of the ventricles which in turn results in the destruction of the striatum (caudate & putamen) - CAUDATE NUCLEUS in particular

186
Q

What are the symptoms of Huntington’s Disease?

A

Decreased muscle tone
• Overshooting movements
Dementia & personality change

187
Q

What is the treatment for Huntington’s Disease?

A

Treated with dopamine receptor BLOCKERS